Master limit 1

Calculus Level 2

lim ( x , y ) ( 0 , 0 ) x sin ( y ) y sin ( x ) x 2 + y 2 = ? \large \displaystyle\lim \limits_{(x,y) \to (0,0) }\frac{x \sin (y) - y \sin ( x)}{x^{2}+ y^{2}} = \ ?


The answer is 0.

This section requires Javascript.
You are seeing this because something didn't load right. We suggest you, (a) try refreshing the page, (b) enabling javascript if it is disabled on your browser and, finally, (c) loading the non-javascript version of this page . We're sorry about the hassle.

4 solutions

Isaac Buckley
Jul 23, 2015

By changing to polar coordinates we can evaluate this.

Making the subs: x = r cos ( θ ) , y = r sin ( θ ) x 2 + y 2 = r 2 x=r\cos(\theta),\,y=r\sin(\theta)\implies x^2+y^2=r^2

Which changes the limit to: lim r 0 cos ( θ ) sin ( r sin ( θ ) ) sin ( θ ) sin ( r cos ( θ ) ) r \lim_{r\to0}\frac{\cos(\theta)\sin(r\sin(\theta))-\sin(\theta)\sin(r\cos(\theta))}{r}

Using L'Hôpital's rule we find:

lim r 0 cos ( θ ) sin ( θ ) ( cos ( r sin ( θ ) ) cos ( r cos ( θ ) ) ) 1 = 0 \lim_{r\to0}\frac{\cos(\theta)\sin(\theta)\left(\cos(r\sin(\theta))-\cos(r\cos(\theta))\right)}{1}= 0

Shashank Ojha
Jul 25, 2015

Jun!!!. It wasn't solve in this way. Follow Isaac solution or you can solve it also by using inequalities

Agustinus Law
Jul 24, 2015

Intuition

After imagining the overall behavior of the function by imagining the values at extreme values, at special cases y = ± x y=\pm x and varying one variable while the other is a constant. I immediately became convinced it is 0.

To prove it

I split the limit into two terms lim ( x , y ) ( 0 , 0 ) sin ( y ) x + y 2 / x \lim_{(x,y)\to (0,0)} \frac{\sin (y)}{x+y^2/x} regarded x x as a constant taking the limit as y 0 y \to 0 then it's always zero. The other term has the same result by analogy and symmetry. Therefore the limit is always zero.

Improvement and corrections on my thought processes are always appreciated.

Jun Arro Estrella
Jul 23, 2015

obviously it is zero since (x,y) has the same value..

Can you please elaborate on what you mean?

Are you assuming the limit exists and then taking the limit with x = y x=y ?

Isaac Buckley - 5 years, 10 months ago

Jun!!!. It wasn't solve in this way. Follow Isaac solution or you can solve it also by using inequalities

Refaat M. Sayed - 5 years, 10 months ago

0 pending reports

×

Problem Loading...

Note Loading...

Set Loading...